1. A survey conducted by a national research center asked a random sample of 920
teenagers in the United States how often they use a video streaming service.
From the sample, 59% answered that they use a video streaming service every day.
a. Construct and interpret a 95% confidence interval for the proportion of all
teenagers in the United States who would respond that they use a video
streaming service every day.
b. Based on the confidence interval in part (a), do the sample data provide
convincing statistical evidence that the proportion of all teenagers in the United
States who would respond that they use a video streaming service every day is
not 0.5? Justify your answer.


Mean Standard Deviation Sample Size
Standard care 0.57 0.26 56
New treatment 0.69 0.27 56

2. Patients experiencing symptoms of a heart attack are routinely transported to a
hospital in an ambulance. In a study of a new treatment thought to reduce damage to
the heart, patients experiencing symptoms of a heart attack were randomly assigned to
one of two groups. During transportation to the hospital, patients in one group received
standard care, and patients in the other group received the new treatment consisting of
standard care and the application of a blood pressure cuff.
The response variable measured for each patient was a number between 0 and 1,
referred to as the myocardial salvage index (MSI). A higher MSI value indicates a more
positive outcome for the patient. Summary statistics for the MSI responses of the two
groups are shown in the table.
Do the data provide convincing statistical evidence that the new treatment results in a
higher mean MSI value than does the standard care among people similar to the
patients in the study?

Answers

Answer 1

Since the calculated t-value is greater than the critical t-value, we reject the null hypothesis, concluding that there's convincing evidence that the new treatment results in a higher mean MSI value than standard care.

How to solve

a. The 95% confidence interval for the proportion of teenagers using video streaming services daily is (0.5579, 0.6221). This means we're 95% confident that 55.79% to 62.21% of US teenagers use such services daily.

b. The results demonstrate that the proportion is not 0.5 because the full confidence interval is higher than 0.5.2. We obtain a t-value of roughly 2.51 with 108.52 degrees of freedom using a two-sample t-test.

At a significance level of 0.05, the critical t-value for a one-tailed test is approximately 1.66.

Since the calculated t-value is greater than the critical t-value, we reject the null hypothesis, concluding that there's convincing evidence that the new treatment results in a higher mean MSI value than standard care.

Read more about confidence interval here:

https://brainly.com/question/15712887

#SPJ1


Related Questions

How large a sample is needed in order to be 90% confident that the sample proportion will not differ from the true proportion by more than 3%? a previous study indicates that the proportion is 21%

Answers

Using the data given such as the margin of error, sample size and z-score, the maximum sample size required is 499

How large a sample is needed in order to be 90% confident that the sample proportion will not differ from the true proportion by more than 3%?

To solve this problem, we need to know the minimum sample size required. The formula for this is given as;

n = (z² * p(1 - p))/ E²

n = sample sizeE = Maximum margin of errorp = estimated proportion from the given previous studyz = z-score

Substituting the values into the formula;

n = (1.645² * 0.21(1 - 0.21))/ 0.03²

n = 498.8

The sample size should be at least approximately 499

Learn more on sample size here;

https://brainly.com/question/28583871

#SPJ1

express 7 min 30 sec. as a percentage of 1 hour​

Answers

Answer:

12.5 is the answer

Step-by-step explanation:

7 × 60 ( 7 being the minutes, 60 being the hour )

= 420

+ 30

= 450

450 ÷ 3600 × 100

45000 ÷ 3600

450 ÷ 30

= 12.5

12.5 being the answer

Answer: 12.5%

Hope this helps! Good luck!

Which of the following numbers is shown plotted on the number line?
0
0 2.75 0 2 0-3 0-2 0-15

Answers

Answer:

-3 3/8

Step-by-step explanation:

The only number we're given on the number line is 0, but we can assume that each tick on the number line represents 1 units.  We see that between the ticks that are 4 and 3 spaces away from 0.Since the point is to the left of 0, we know the number must be negative.The only answer that would make sense is -3 3/8, since the tick 4 places away from 0 is -4 and the tick 3 units away from 0 is -3 and -3 3/8 lies between -3 and -4

At which values in the interval [0, 2π) will the functions f (x) = cos 2x + 1 and g(x) = sin x + 2 intersect?

x equals 0 comma pi over 6 comma 5 times pi over 6 comma pi
x equals 0 comma pi comma 7 times pi over 6 comma 11 times pi over 6
x equals pi over 2 comma 7 times pi over 6 comma 3 times pi over 2 comma 11 times pi over 6
x equals pi over 6 comma pi over 2 comma 5 times pi over 6 comma 3 times pi over 2

Answers

The functions f(x) = cos 2x + 1 and g(x) = sin x + 2 intersect at x = pi/6, pi/2, 5pi/6, and 3pi/2 in the interval [0, 2π).

To find the intersection points, we set f(x) equal to g(x) and solve for x.

cos 2x + 1 = sin x + 2

Rearranging, we get:

cos 2x = sin x + 1

Using the identity cos 2x = 1 - 2sin²x, we can rewrite this as:

1 - 2sin²x = sin x + 1

Simplifying and factoring, we get:

2sin²x + sin x = 0

sin x(2sin x + 1) = 0

Therefore, sin x = 0 or sin x = -1/2. Solving for x in the interval [0, 2π), we get x = pi/6, pi/2, 5pi/6, and 3pi/2. These are the values at which f(x) and g(x) intersect.

learn more about intersection points here:

https://brainly.com/question/29188411

#SPJ1

please answer the question in pic

Answers

Note that given the factors above, Orr should replace the above plant. This has to do with depreciation and book value.

Why is this so  ?

We need to first compute the current total cost of keeping the old plant.

Book value = $4,500,000 - $1,000,000/year x 4 years

= $500,000

Thi smean that the current total cost of keeping the old plan tfor the reamining 10 year is

Total cost of keeping old plant = $1,000,000/year x 10 years - $500,000 + $150,000

= $9 650 000

Now lets calculate the total cost of purchasing the new plant for 10 years.

$600,000 - $150,000

= $450,000 per year.

The salvage value after 10 years is $0.

Total cost of new plant = $3,500,000 + $450,000/year x 10 years = $8,000,000

Since the cost of keeping is higher than the new one, then Orr should get a new plant.

$9 650 000 > $8,000,000

Learn more about depreciation:
https://brainly.com/question/30531944
#SPJ1

If it takes 15 minutes for water in a pot to boil, the expression 20x+ 15 can be used to find the total time needed to cook x batches of boiled cornbread. Which expression can also be used to determine the total time needed to cook x batches of boiled cornbread? PLS ANSWER ASAPP I REALLY NEED TO ANSWER THIS QUESTION

A 18x + 4x + 6 + 7
B 10x + 10x + 20 - 5
C 6(5x + 5) - 10x + 15
D 3(x + x + x + 3)+ 2x + 15

Answers

the correct expression which can also be used to determine the total time needed to cook x batches of boiled cornbread is,

B) 10x + 10x + 20 - 5.

Given that;

The expression 20x + 15 represents the total time needed to cook x batches of boiled cornbread.

We can simplify this expression by factoring out the common factor of 5: = 20x + 15

= 5(4x + 3)

Therefore, the expression that can also be used to determine the total time needed to cook x batches of boiled cornbread must have a factor of 5 in it.

Option A:

18x + 4x + 6 + 7 does not have a factor of 5,

so it cannot be the correct answer.

Option B:

10x + 10x + 20 - 5

= 20x + 15,

which is the same as the original expression.

Therefore, Option B is equivalent to the original expression and can also be used to determine the total time needed to cook x batches of boiled cornbread.

Option C:

6(5x + 5) - 10x + 15 = 30x + 15 - 10x + 15 = 20x + 30,

which does not have a factor of 5.

Therefore, Option C is not correct.

Option D:

3(x + x + x + 3) + 2x + 15 = 9x + 6 + 2x + 15 = 11x + 21,

which does not have a factor of 5.

Therefore, Option D is not correct.

Therefore, the correct answer is,

B) 10x + 10x + 20 - 5.

Learn more about the mathematical expression visit:

brainly.com/question/1859113

#SPJ1

100 POINTS

Question:

Answers

read the paragraph and then you’d have all the information that you need

Answer:

First braclets coast 3$ and necklaces coast 6$ so now in the box you can put 3in the first one then 6 in the next and keep going its like multiples of 3 and 6 :) Hope this helps.

Step-by-step explanation:

 Cooper can purchase a item for $45.00, Which store has the best buy?

A. Store 1: 25% off coupon
B. Store 2: $15 dollars off
C. Store 3: 10% off

I’ll give brainly please help me

Answers

Store 2 has the best price

Please solve this for me!

Answers

Answer: 1; 520/9 number 2: 9/2 number 3: d number 4:d number 5: 151.7

Step-by-step explanation:

The value of x in the following equation 4x-5=7 is .........​

Answers

Answer:

Step-by-step explanation:

4x - 5 = 7

4x = 7 + 5

4x = 12

x = 12/4

x = 3

Given the two concentric circles with center X below, find the area of the shaded region. Round your answer to the nearest tenth if necessary.

Answers

The area of the shaded region is given as follows:

298.3 units².

How to calculate the area of a circle?

The area of a circle of radius r is given by the multiplication of π and the radius squared, as follows:

A = πr²

The larger circle has a radius of 7 + 5 = 12 units, hence it's area is given as follows:

A = 3.14 x 12²

A = 452.16 units².

The smaller circle has a radius of 7 units, hence it's area is given as follows:

A = 3.14 x 7²

A = 153.86 units².

Hence the area of the shaded region is given as follows:

452.16 - 153.86 = 298.3 units².

More can be learned about the area of a circle at https://brainly.com/question/15673093

#SPJ1

When a retired police officer passes away, he leaves $300,000 to be divided among his three children and six grandchildren. The will specifies that each child is to get twice as much as each grandchild. How much does each get?

Answers

Answer:

so first off if its six grandchildren and 3 children then that means 9 people but if the children get double the amount distributed then lets treat it like 12 people so 25,000 each meaning each child of the police officer gets 50 thousand and each grandchild gets 25 thousand

Step-by-step explanation:

hope this helps !

Each child will get 50,000 and each grandchild will get 25,000

If the number 48700 is written in scientific notation, it will be in the form 4.87×10n. What is the value of n?

Answers

If the number 48700 is written in scientific notation, it will be in form 4.87×10ⁿ, and the value of n is 4.

What is scientific notation?

Scientific notation describes the shorthand way of writing very large or very small numbers in standard forms.

Writing in scientific notation, a number between 1 and 10 is multiplied by a power of 10.

48700 = 4.87×10ⁿ = 4.87×10⁴

Thus, the n represents the exponent of power 10 to which the number is reduced.

Learn more about scientific notations at https://brainly.com/question/27862246.

#SPJ1

50 Points! Algebra question. Photo attached. Please show as much work as possible. Thank you!

Answers

Answer:

the answer for this question is x=8

A study of the consultants in a particular industry has determined that the standard deviation of the hourly fee of the consultants is $23. A random sample of
100 consultants in the industry has a mean hourly fee of $113. Find a 99% confidence interval for the true mean hourly fee of all consultants in the industry.
Then give its lower limit and upper limit.
Carry your intermediate computations to at least three decimal places. Round your answers to one decimal place. (If necessary, consult a list of formulas.)
Lower limit:
Upper limit:
X
5

Answers

If a study of the consultants in a particular industry has determined that the standard deviation of the hourly fee of the consultants is $23.  the 99% confidence interval for the true mean hourly fee of all consultants in the industry is (107.1, 118.9).

How to find the confidence interval?

The formula for a confidence interval for the population mean is:

CI = X- bar ± z* (σ/√n)

where:

X- bar  = sample mean

z* = z-score for the desired level of confidence (99% in this case)

σ = population standard deviation

n = sample size

Substituting the given values, we get:

CI = 113 ± 2.576 * (23/√100)

Calculating the standard error:

SE = σ/√n = 23/√100 = 2.3

Now we can substitute this value into the formula:

CI = 113 ± 2.576 * 2.3

CI = 113 ± 5.92

So,

Lower limit: 107.1

Upper limit: 118.9

Therefore, the 99% confidence interval for the true mean hourly fee of all consultants in the industry is (107.1, 118.9).

Learn more about confidence interval here:https://brainly.com/question/15712887

#SPJ1

The function h(t) = 4 + 64t – 16t2 models the height h, in feet, of a ball thrown in the air, after t seconds.

Part A

What is the vertex of the graph of the function, (t, h(t))?

(
,
)

Part B

What does the t-coordinate of the vertex represent?
A. the ball's maximum height
B. the time it takes for the ball to reach its maximum height
C. the time it takes for the ball to hit the ground
D. the height the ball was thrown from
Part C

What does the h(t)-coordinate of the vertex represent?
A. the ball's maximum height
B. the time it takes for the ball to reach its maximum height
C. the time it takes for the ball to hit the ground
D. the height the ball was thrown from

Answers

Part A) The vertex of the graph of the function, (t, h(t)) is (2, 68).

Part B) The t-coordinate of the vertex represent is the time it takes for the ball to reach its maximum height (option b)

Part C) The h(t)-coordinate of the vertex represent is the ball's maximum height (option a).

Part A asks for the vertex of the graph of the function, which is the point where the function reaches its maximum or minimum value. To find the vertex of a quadratic function like

=> h(t) = 4 + 64t – 16t²,

we can use the formula

=>  t = -b/2a,

where a, b, and c are the coefficients of the quadratic equation ax² + bx + c and t is the input variable (in this case, the time).

The t-coordinate of the vertex is simply the value we get when we plug this formula into our equation.

So, for

=> h(t) = 4 + 64t – 16t²,

we have a = -16, b = 64, and c = 4.

Plugging these values into the formula

=> t = -b/2a,

we get

=> t = -64/(2*(-16)) = 2.

The t-coordinate of the vertex is therefore 2.

To find the h(t)-coordinate of the vertex, we can simply plug t = 2 into the function h(t) = 4 + 64t – 16t² and evaluate it.

This gives us

=> h(2) = 4 + 64(2) – 16(2²) = 68.

Therefore, the vertex of the graph of h(t) is (2, 68).

Part B asks what the t-coordinate of the vertex represents. We know that the t-coordinate is the time at which the ball reaches its maximum height.

Therefore, the correct answer is B: the time it takes for the ball to reach its maximum height.

Part C asks what the h(t)-coordinate of the vertex represents. We just found that the h(t)-coordinate of the vertex is the maximum height of the ball.

Therefore, the correct answer is A: the ball's maximum height.

To know more about function here

https://brainly.com/question/28193995

#SPJ1

Boyd's board shop grt a shipment of 92 new skateboards wheels. Boyd assembles the skateboards and puts 4 of the new wheels on each board. You can use a function to describe the number of new wheels Boyd has left after assembling x skateboards

Answers

The function to describe the number of new wheels Boyd has left is f(x) = 92 - 4x


Calculating the function to describe the number of new wheels Boyd has left

From the question, we have the following parameters that can be used in our computation:

Shipment of 92 new skateboards wheels. 4 of the new wheels on each board.

After assembling x skateboards, we have

f(x) = Shipmnet - Rate * x

Substitute the known values in the above equation, so, we have the following representation

f(x) = 92 - 4 * x

Evaluate

f(x) = 92 - 4x

Hence, the function is f(x) = 92 - 4x

Read more abiut linear relation at

https://brainly.com/question/30318449

#SPJ1

Jackson's Ice Cream Shop must have a mean of 110 visitors per day in order to make a profit. The table below shows the number of visitors during 1 week.

Day of the week | No of Visitors

Sunday.................... 63

Monday.................. 77

Tuesday.................. 121

Wednesday............... 96

Thursday .......................137

Friday................................ 154


Based on the table, how many visitors will need to go to Jackson's Ice Cream shop on Saturday in order for the company to make a profit for the week?


A-108

B-110

C-122

D-222


Please add work/ explain!!

Answers

To find out how many visitors Jackson's Ice Cream Shop needs on Saturday to make a profit for the week, we need to first calculate the total number of visitors for the six days already given in the table. Then we can find out the number of visitors needed on Saturday to reach the required mean of 110 visitors per day for the week.

Total number of visitors for the six days = 63 + 77 + 121 + 96 + 137 + 154 = 648

To make a profit for the week, the total number of visitors for the entire week needs to be:

110 visitors/day × 7 days/week = 770 visitors/week

We already have 648 visitors for the six days, so the number of visitors needed on Saturday to make a profit can be found by subtracting the total number of visitors already counted from the total number of visitors needed for the week:

770 - 648 = 122

Therefore, the answer is (C) 122 visitors on Saturday are needed to make a profit for the week.

PLS HURRY I AM GIVING BRAINLIEST!!!
the question is in the photo!!

Answers

The sum of both sides is 2x² + 4. The length of the third side is 5x³ - 4x² + 3x - 12

What is a triangle?

A triangle is a polygon that has three sides and three angles. Types of triangles are isosceles, right angled, scalene, equilateral.

Given that the perimeter of the triangle is 5x³ - 2x² + 3x - 8. The length of side one is 3x² - 4x - 1 and that of side two is -x² + 4x + 5

The sum of side one and side two is:

Sum = side 1 + side 2 = (3x² - 4x - 1) + (-x² + 4x + 5) = 2x² + 4

The sum of both sides is 2x² + 4

The third side is:

Third side = Perimeter - Sum of two sides = (5x³ - 2x² + 3x - 8) - (2x² + 4) = 5x³ - 4x² + 3x - 12

The length of the third side is 5x³ - 4x² + 3x - 12

Find out more on triangle at: https://brainly.com/question/28470545

#SPJ1

please help with this question! ​

Answers

Answer:

The true statements:

6 is an element of D

12 is an element of D

How much money should Be deposited today in an account that earns 10.5 percent compounded monthly so that it will accumulate to $22,000 in four years

Answers

Answer:

$12,276.24

Step-by-step explanation:

22000 / (1 + 0.105/12)^4*12

12276.24

We can use the formula for compound interest to solve this problem:

A = P(1 + r/n)^(nt)

where A is the future value, P is the present value, r is the annual interest rate, n is the number of times the interest is compounded per year, and t is the number of years.

In this case, we want to find the present value P that will accumulate to $22,000 in four years at an annual interest rate of 10.5% compounded monthly (i.e., n = 12). Therefore, we have:

A = $22,000
r = 0.105/12 (monthly interest rate)
n = 12
t = 4

Substituting these values into the formula and solving for P, we get:

P = A/(1 + r/n)^(nt) = $22,000/(1 + 0.105/12)^(12*4) ≈ $14,005.70

Therefore, the amount that should be deposited today in the account is approximately $14,005.70.

What did Maleuvre write about Gauguin's views on Polynesian women?

Answers

Answer:

Step-by-step explanation:

Observation of the real life and way of life of the peoples of Oceania are intertwined in them with local myths.

Asynchronous activity

Answers

In the triangles given the analysis for same is given below

What is the analysis of the two triangles given above?

1) the corresponding vertices are

R ~ C and

E ~ 0

2) The corresponding angle are

∠E ~∠O  and

∠ R ~ ∠C

3)

The corresponding sides are;

ER ~ CO;
MO ~ EM
CM ~ RM

4)  the congruent angles are:

∠REM ≅ ∠MOC
∠ERM ≅ ∠MCO
∠CMO ≅ EMR

5) the congruent sides are:

ER ≅ CO;
MO ≅ EM
CM ≅ RM

6) the congruent triangles are;

ΔMER ≅ ΔMOC

Learn more about triangles:
https://brainly.com/question/2773823
#SPJ1

The administrator of a county's museum is considering increasing the price of an annual pass by 1% from $25 to $25.25. At the current price, the museum sells 2500 annual passes and the point of elasticity is 0.73. What effect would the increase in price have on the museum's annual pass revenue?

Answers

Answer: the increase in price from $25 to $25.25 would result in a small increase in annual pass revenue of $178.50 or about 0.3%.

Step-by-step explanation: we can calculate the modern yearly pass income by increasing the modern amount requested by the unused cost of $25.25:

Unused yearly pass income = Modern cost x Modern amount requested

Unused yearly pass income = $25.25 x 2482 = $62,678.50

We as of now know that the ancient yearly pass income is $62,500 (since 2500 passes were sold at $25 each), so ready to calculate the alter in yearly pass income:

Alter in yearly pass income = Unused yearly pass income - Ancient yearly pass income

Alter in yearly pass income = $62,678.50 - $62,500 = $178.50

1. A rectangular block of wood has dimensions 24 cm by 9 cm by 7 cm. It is cut up into children's bricks. Each brick is a cube of side 3 cm.
(a) Find the largest number of bricks that can be cut from the block.
(b) Find the volume of wood that is left. ​

Answers

The dimensions of the rectangular block indicates;

(a) The largest number of bricks that can be cut from the block is 48 bricks

(b) The volume of the wood left is 216 cm³

What is the volume of a rectangular block?

The volume of the rectangular block is the product of the length, L, width, W, and height, H.

1. The dimensions of the rectangular block indicates that the block volume is; V = 24 cm × 9 cm × 7 cm = 1,512 cm³

The largest number of bricks that have side lengths of 3 cm that can be cut from the block is therefore;

Number of cubes; (24)/3 × (9)/3 × (7)/3 ⇒ 8 × 3 × 2 = 48

48 cube bricks can be cut from the block

(b) The volume of wood left = 1512 - 48 × 3 × 3 × 3 = 216

The volume of wood left is 216 cm³

Learn more on the volume of regular solids here: https://brainly.com/question/31010344

#SPJ1

A certain test preparation course is designed to help students improve their scores on the LSAT exam. A mock exam is given at the beginning and end of the course to determine the effectiveness of the course. The following measurements are the net change in 5 students' scores on the exam after completing the course:

−5,19,4,0,1
Using these data, construct a 80% confidence interval for the average net change in a student's score after completing the course. Assume the population is approximately normal.


Step 3 of 4 : Find the critical value that should be used in constructing the confidence interval. Round your answer to three decimal places.

Answers

The critical value for an 80% confidence interval is 1.28 (rounded to three decimal places).

What is the statistics?

Statistics is a branch of mathematics that involves the collection, analysis, interpretation, presentation, and organization of data.

To find the critical value for an 80% confidence interval, we need to determine the z-score corresponding to the level of confidence. We can use a standard normal distribution table or a calculator to find this value.

Using a standard normal distribution table, we can find the z-score that corresponds to an 80% confidence level by finding the area between the mean and the upper tail of 10%, which is (100% - 80%)/2 = 10%.

The area between the mean and 10% is 0.5 - 0.10 = 0.40. Using the standard normal distribution table, the z-score that corresponds to an area of 0.40 is approximately 1.28.

Hence, the critical value for an 80% confidence interval is 1.28 (rounded to three decimal places).

To learn more about z-score, Visit

https://brainly.com/question/15222372

#SPJ1

A political pollster wants to know what proportion of voters are planning to
vote for the incumbent candidate in an upcoming election. A poll of 150
randomly selected voters is taken from the more than 2,000 voters in the
population, and 78 of those selected plan to vote for the incumbent
candidate.
Based on this sample, which of the following is a 90% confidence interval
for the proportion of all voters who plan to vote for the incumbent
candidate?
Choose 1 answer:
780.067
78±0.080
0.520.067

Answers

The confidence interval is 0.52±0.067.(option c)

What is confidence interval?

In statistics,  confidence interval means that the probability in which a population parameter will fall between a set of values for a certain proportion of times.

A political pollster wants to know what proportion of voters are planning to

vote for the incumbent candidate in an upcoming election. A poll of 150

randomly selected voters is taken from the more than 2,000 voters in the

population, and 78 of those selected plan to vote for the incumbent

candidate.

The formula for confidence interval(CI) is given by:

                      CI=  p ± z √{p(1-p)/n} ----------------(1)

Here from the given data p= 78/150= 0.52

                                          n= 150

For 90% confidence interval the value of z= 1.645.

Putting all the values in equation (1) we get,

                         CI= 0.52± 1.645√{(0.52×0.48)/150}

                             = 0.52±1.645×0.04

                             = 0.52±0.067

Hence, the confidence interval is 0.52±0.067.

To know more about confidence interval

https://brainly.com/question/20309162      from the link.

#SPJ9

The Morning Gazette offers employees 1.65% of the average of their last 3 years of annual compensation for each year of service. Rita began working for the Morning Gazette in 1994. She retired in 2016. In 2014, she made $76,000 per year. Thereafter, she received a 3% salary increase each year until she retired.
a) How much did she earn for each year from 2014 through 2016?
b) What is the average of her last five years of working?
c) How much was his annual retirement benefit?

Answers

She earn fοr each year frοm 2014 thrοugh 2016 is 80628.40. The average οf her last five years οf wοrking $78,302.80. His annual retirement benefit was $28,423.92.

a) Salary οf 2014 : $76000

Salary in 2014 is the salary in 2015 increased by 3% οf the salary

= $76,000+3%

Salary οf 2015 = $78,280

Salary in 2015 is the salary in 2016 increased by 3% οf the salary

=$78,280+3%

Salary οf 2016 = $80,628.40

Hence, she earn fοr each year frοm 2014 thrοugh 2016 is 80628.40

b) The average οf the last three years is the sum οf the salaries divided by the number οf salaries.

= $76,000+$78,280+$80,628.40 / 3

= $78,302.80

Hence, the average οf her last five years οf wοrking $78,302.80

c ) The annual retirement benefit is the prοduct οf the rate and the average and the number οf years οf service.

= 1.65%×$78,302.80×22

=$28,423.92

Hence,  his annual retirement benefit was $28,423.92.

To know more about salary check below link:

https://brainly.com/question/28920245

#SPJ1

3.4 MIXED FACTORING

1. Utilize all of the strategies for factoring in order to factor the following polynomials.
Reminder: Combine like-terms prior to factoring.

(2x^2 + 23x) - (3x - 18)

Answers

Answer:

2 (x + 1)(x + 9)

Step-by-step explanation:

The given expression is
[tex]\left(2x^2\:+\:23x\right)\:-\:\left(3x\:-\:18\right)[/tex]

and we are asked to factor it

Step 1
Remove parentheses
[tex]\left(2x^2\:+\:23x\right) = 2x^2 + 23x\\\\-\:\left(3x\:-\:18\right) = - (3x) - (-18) = -3x + 18\\\\[/tex]

Step 2
Add the individual terms to correspond to the original expression:
[tex]\left(2x^2\:+\:23x\right)\:-\:\left(3x\:-\:18\right) \\\\= 2x^2 + 23x -3x + 18\\\\= 2x^2 + 20x + 18[/tex]

Step 3
Factor out common term 2

[tex]2x^2+20x+18 \\\\= 2\left(x^2+10x+9\right)[/tex]

Step 4
Factor [tex]x^2+10x+9[/tex]

To do this find two numbers such that their sum = 10 and product = 9
We can easily see that these two numbers are 1 and 9 because 1 + 9 = 10 and 1 x 9 = 9

Therefore, splitting the expression into groups we get
[tex]x^2 + 10x + 9 = x^2 + 1x + 9x + 9\\\\[/tex]

Step 5

Factor:

[tex]x^2 + 1x = x(x+ 1)\\9x + 9 = 9(x + 1)\\\\[/tex]

Therefore
[tex]x^2 + 1x + 9x + 9 = x(x + 1) + 9(x + 1)\\\\[/tex]

Step 6

Factor common term (x + 1) from the expression
[tex]x(x + 1) + 9(x + 1) = (x + 1)(x + 9)[/tex]

Step 7
Putting it all together
Remember we factored out the 2 in step 3 so we got to put it back into the factored expression giving the final factored expression as

[tex]2 (x + 1)(x + 9)[/tex]

Write an equation that represents the line.

Answers

Answer:

y = (3/4)x + 2

Step-by-step explanation:

A line (or linear function) can be represented by the equation y = mx + b where "m" is the slope and "b" is the y-intercept.

The y-intercept is when x = 0 or when the line crosses the y-axis. In the graph, notice that the line crosses at (0, 2). Thus, the y-intercept is at y = 2 and "b" must be "2".

Next, we have to find the slope, "m". The slope is defined as a change in "y" divided by a change in "x". Lets use the two points which are marked on the graph already, (4, 5) and (0, 2). We first subtract the y-values, and then divide that by the difference between corresponding x-values:

(5 - 2)/(4 - 0) = 3/4                           The slope is 3/4, so m = 3/4

Now, plug "m" and "b" into the equation:

y = (3/4)x + 2

Answer:

y=(3/4)x+2

Step-by-step explanation:

So first you need to find the slope (3/4). To find that you can use the rise over run method. In this case they have already chosen two points on the line, but you could use any exact points. Count how many up from 2 to 5 (in green), which is 3 then how many across (in red) which is 4. Then put the rise (3) over the run (4) so 3/4.

Then b (2) is equal to y in the y intercept which is (0,2)

Other Questions
What is the volume of a can with a diameter of 4 inches and a height of 9 inchesand what is the surface area of a can with a diameter of 4 inches and height of 9 inchesand ratio of surface area to volume Readingproportional relationships - part 1do the values in the table represent a proportional relationship?012.3y0356select from the drop-down menu to correctly complete the statement.all of the y-values choose...a constant multiple of the corresponding x-values, so the relationship choose...12345678910next The gleaners by jean-francois millet. Three poor women search a field for food after a harvest. What type of atmosphere is depicted in the image above, and how did the artist achieve this? Research and briefly summarize the following savings options. In your summary, make sure you mention the level of risk associated with each option and recent returns on investment. Savings account Certificates of deposit Stocks Bonds Mutual funds Employer sponsored savings plan Physical assets Commodities Write a diary about the blue factory it has to be 500-600 words A runner takes 4. 92 seconds to complete a sprint. If they run the sprint 19 times, how many total seconds would it take? Find the open intervals where the function is concave upward or concave downward. Find any inflection points. f(x) = 11x - 11 e - Where is the function concave upward and where is it concave downward? can someone help me on this , and sure not if there correct Choose the correct statement that best distinguishes between astrology and astronomy. AAstrology is the study of stars only. Astronomy is the study of planets only.BAstrology is the study of the Moon only. Astronomy is the study of stars and planets only.CAstrology is the study of influence of cosmic objects on human lives. Astronomy studies celestial objects and related phenomena.DAstrology is the study of planets only. Astronomy is the study of stars only. Taub went shopping for a new phone. To find the total plus tax, she multiplied the price of the phone by 1.055. What percent tax did she pay? Teleconferencing allows workers from around the world to the czech republic has a gdp of 2,000 billion koruny. the exchange rate is 20 koruny per u.s. dollar. the czech population is 20 million. calculate the per person gdp of the czech republic in u.s. dollars. 1. A ___ can help to indicate an "endless loop," or a continual process without progression. A. scatter diagram B. range control chart C. flow chart D. normal distribution curve2. Which is not an achievable goal of process improvement?A. Identifying sources of variationB. Eliminating common causes of variationC. Eliminating assignable causes of variation D. Measuring the amount of variation 1.a capital investment is expected to achieve long-term benefits for the organization thatgenerally fall into three categories: financial benefits, nonfinancial returns and the ability toattract more funds in the future.true or false? the area of a rectangular sticker is 105 square centimeters. the perimeter is 44 centimeters. what are the dimensions of the sticker? How does Mr. Peters change throughout the story? What evidence does the author include? Answer the following questions for the functionf(x) = x^3/x^2 - 4 defined on the interval (18, 19) Enter the z-coordinates of the vertical asymptotes of f(x) as a comma-separated list. That is, if there is just one value, give it; if there are more than one, enter them separated commas; and if there are nono, enter NONE Use figure 7.3.1 to answer the following questions. What is the name of this branched alkene? Please help me as fast as possible I need to study, please! liam is known by others as a rebellious person who seems to have a complete disregard for rules and laws. he does what he wants, without regard for the consequences and without a sense of guilt. some accuse him of being emotionally shallow. he would probably score high on the mmpi-2 scale called: